Đến nội dung

tuananh2000 nội dung

Có 216 mục bởi tuananh2000 (Tìm giới hạn từ 28-04-2020)



Sắp theo                Sắp xếp  

#679044 Tìm tất cả các giá trị $m$ sao cho trên đồ thị $(C)$ tồn...

Đã gửi bởi tuananh2000 on 30-04-2017 - 18:19 trong Hàm số - Đạo hàm

Bài 1: Cho hàm số $y=\frac{2x}{x+1} (C)$. Tìm tọa độ $M(a;b)$ có hoành độ $a$ dương thuộc $(C)$, biết tiếp tuyển của $(C)$ tại $M$ cắt $2$ trục $Ox;Oy$ tại $2$ điểm $A$ và $B$ sao cho tam giác $ABO$ có diện tích $\frac{1}{4}$. Tìm mối liên hệ giữa $a$ và $b$

Bài 2: Cho hàm số $(C):y=\frac{2x}{x+2}$. Viết phương trình tiếp tuyến với $(C)$, biết rằng khoảng cách từ $I(-2;2)$ đến tiếp tuyến là lớn nhất

Bài 3: Cho hàm số: $(C):y=\frac{2x+3}{x+2}$. Tìm những điểm $M$ thuộc $(C)$ sao cho tiếp tuyến tại $M$ cắt hai đường thẳng $x=-2, y=2$ lần lượt tại $A,B$ sao cho vòng tròn ngoại tiếp tam giác $IAB$ có bán kính nhỏ nhất với $I(-2;2)$.

Bài 4: Cho hs: $y=\frac{1}{3}mx^{3}+(m-1)x^{2}+(4-3m)x+1$ có đồ thị $(C)$, Tìm tất cả các giá trị $m$ sao cho trên đồ thị $(C)$ tồn tại duy nhất một điểm $A$ có hoành độ âm mà tiếp tuyến với $(C)$ tại $A$ vuông góc với đường thẳng : $x+2y-3=0$




#675520 Chứng minh phương trình có $4$ nghiệm phân biệt $x_{i...

Đã gửi bởi tuananh2000 on 28-03-2017 - 14:23 trong Hàm số - Đạo hàm

Chứng minh phương trình có $4$ nghiệm phân biệt $x_{i};i=\overline{1,4}$ 

$f(x)=2x^{4}-6x^{3}-10x^{2}+5x+3=0$ và hãy tính tổng $S=\sum_{i=1}^{4}\frac{2x_{i}^{2}-1}{(x_{i}-1)^{2}}$

 




#672018 Tính $lim(n.\sqrt{n^{2}+n}-\sqrt{n^...

Đã gửi bởi tuananh2000 on 18-02-2017 - 21:26 trong Dãy số - Giới hạn

Tính $lim(n.\sqrt{n^{2}+n}-\sqrt{n^{2}+n+1}.\sqrt[3]{n^{3}+n})$




#668537 Tìm $a$ để $(v_{n})$ là $CSN$, suy ra...

Đã gửi bởi tuananh2000 on 16-01-2017 - 17:03 trong Dãy số - Giới hạn

$1)$ Cho $(u_{n}):u_{1}=5;u_{n+1}=3u_{n}-4$. Đặt $v_{n}=u_{n}+a$ với mọi $n$ lớn hơn hoặc bằng $1$. Tìm $a$ để $(v_{n})$ là $CSN$, suy ra số hạng tổng quát của $(u_{n})$

$2)$ Cho $(u_{n}):u_{1}=2;u_{n+1}=\frac{u_{n}}{2u_{n}+5}$.Tìm số hạng tổng quát của $(u_{n})$

$3)$ Cho $(u_{n}):u_{1}=\frac{1}{3},u_{n+1}=\frac{n+1}{3n}.u_{n}$ với mọi $n$ lớn hơn hoặc bằng $1$ 

a) $CMR:$ $(v_{n}):v_{n}=\frac{u_{n}}{n}$ lập thành $CSN$

b) Tính tổng $T=\frac{u_{1}}{1}+\frac{u_{2}}{2}+...+\frac{u_{n}}{n}$




#656546 $\sum sin(\frac{A}{2}).sin(\frac...

Đã gửi bởi tuananh2000 on 03-10-2016 - 10:38 trong Bất đẳng thức và cực trị

Chứng minh bất đẳng thức :

$\sum (sin(\frac{A}{2}).sin(\frac{B}{2}))\leq \frac{1}{6}.(\sum (sin(A).sin(B)))+\frac{3}{8}$ với $A;B;C$ là 3 góc của tam giác




#645939 Nhận và khoe áo đồng phục của Diễn đàn.

Đã gửi bởi tuananh2000 on 22-07-2016 - 00:13 trong Góc giao lưu

Em cũng nhận được áo rồi =)) áo đẹp lắm ạ  :like




#625204 Kết quả TST 2016

Đã gửi bởi tuananh2000 on 05-04-2016 - 21:31 trong Thi HSG Quốc gia và Quốc tế

anh nguyenta98 ( Tạ Hà Nguyên ) rank bao nhiêu vậy mn ? :(




#617983 Topic về phương trình và hệ phương trình

Đã gửi bởi tuananh2000 on 02-03-2016 - 04:29 trong Phương trình - hệ phương trình - bất phương trình

ĐKXĐ: .....

Ta có pt (1) $\Leftrightarrow (5x^2+\frac{3}{2}x-3)-(3x+1)\sqrt{2x^2-1}=0$

$\Leftrightarrow 2(2x^2-1)-(3x+1)\sqrt{2x^2-1}+x^2+\frac{3}{2}x-1=0$

Đặt $\sqrt{2x^2-1}=a (a\geq 0)$ ta có:

$2a^2-(3x+1)a+x^2+\frac{3}{2}x-1=0$

$\Leftrightarrow (a-x+\frac{1}{2})(2a-x-2)=0$

Đến đây dễ rồi 

Từ dòng đỏ trên sao bạn có ý tưởng phân tích thành dòng đỏ dưới được vậy, mình vẫn chưa hiểu rõ cách suy luận của bài toán lắm  :icon6:




#617710 $x^{2}+8x\leq 6\sqrt{x^{3}+3x}+3...

Đã gửi bởi tuananh2000 on 29-02-2016 - 20:37 trong Phương trình - hệ phương trình - bất phương trình

Giải bất phương trình :

1) $x^{2}+8x\leq 6\sqrt{x^{3}+3x}+3$

2) $\frac{3-2\sqrt{x^{2}+3x+2}}{1-2\sqrt{x^{2}-x+1}}>1$




#617705 Topic về phương trình và hệ phương trình

Đã gửi bởi tuananh2000 on 29-02-2016 - 20:29 trong Phương trình - hệ phương trình - bất phương trình

Bài 299 : $\sqrt{(x+2)(2x-1)}-3\sqrt{x+6}=4-\sqrt{(x+6)(2x-1)}+3\sqrt{x+2}$

Bài 300 : $(3x+1)\sqrt{2x^{2}-1}=5x^{2}+\frac{3x}{2}-3$




#614688 Topic về phương trình và hệ phương trình

Đã gửi bởi tuananh2000 on 13-02-2016 - 11:38 trong Phương trình - hệ phương trình - bất phương trình

Bài 208 (trích từ bạn minhminh98 ) , mình không nhớ ở topic này có chưa nhưng thấy khá khó :

 

 $\left\{\begin{matrix}x^2y+x^2+1=2x\sqrt{x^2y+2} & \\ y^3(x^6-1)+3y(x^2-2)+3y^2+4=0 & \end{matrix}\right.$

Mình làm thử bài này thì thấy gặp vấn đề là 

1) Ở dữ kiện đầu có cho $x^2y+x^2+1=2x\sqrt{x^2y+2}$ sau khi xét các TH sẽ được $y=\frac{x^{2}\pm 2x-1}{x^{2}}$

2) Ở dữ kiện sau ta có thể phân tích nhân tử  $y^3(x^6-1)+3y(x^2-2)+3y^2+4=0$ thành $(x^2y-y+1) (x^4y^2+x^2y^2-x^2 y+y^2-2 y+4) = 0$ nhưng vì $x^4y^2+x^2y^2-x^2 y+y^2-2 y+4=0$ có nghiệm khá xấu nên mình cũng chưa biết xử lí như thế nào @@!




#584072 TOPIC Tổ hợp-Xác suất

Đã gửi bởi tuananh2000 on 22-08-2015 - 20:02 trong Tổ hợp - Xác suất và thống kê - Số phức

 

 

@};- $\boxed{5}$Có 3 nam và 3 nữ cần xếp ngồi vào một hàng ghế.Hỏi có mấy cách xếp sao cho:

a)Nam,nữ ngồi xen kẽ

b)Nam,nữ ngồi xen kẽ và có một người nam A,một người nữ B phải ngồi kề nhau?

c)Nam,nữ ngồi kề nhau và có một người nam C,một người nữ D không được ngồi kề nhau?

 

 

a) Ta có thể xếp nam ngồi đầu và nữ ngồi sau theo thứ tự $Na-Nu-Na-Nu-Na-Nu$ hoặc nữ ngồi đầu và nam ngồi sau theo thứ tự $Nu-Na-Nu-Na-Nu-Na$ , vậy có $2.3!.3!=72$ cách

b) Trong trường hợp người $A$ ngồi trước người $B$ theo thứ tự xen kẽ $A-B-Na-Nu-Na-Nu$ thì có $4$ cách chọn , trong đó người nam $A$ có thể ở $5$ vị trí khác nhau nên có $20$ cách . Tương tự với trường hợp người $A$ ngồi sau người $B$ thì có $40$ cách 

c)Trong trường hợp người $C$ ngồi đầu hoặc ngồi cuối theo  thứ tự $C-1-2-3-4-5$ và $2-3-4-5-1-C$ thì người nam $D$ có thể ngồi ở $4$ vị trí $2,3,4,5$ và không cần xen kẽ nên 

Vị trí $C$ có $1$ cách chọn

Vị trí $1$ có $4$ cách chọn ( không phải nữ $D$)

Vị trí $2$ có $4$ cách chọn ( có thể nữ $D$)

Vị trí $3$ có $3$ cách chọn

Vị trí $4$ có $2$ cách chọn

Vị trí $5$ có $1$ cách chọn

nên có $2.4.4.3.2!=192$ cách 

Trong trường hợp người $C$ ngồi ở vị trí $2,3,4,5$ theo thứ tự $1-2-3-4-5-6$ . Giả sử $C$ ngồi ở vị trí $2$ thì $1-C-3-4-5-6$

Vị trí $1$ có $4$ cách chọn ( không có nữ $D$ và nam $C$)

Vị trí $C$ có $1$ cách chọn ( phải là nam)

Vị trí $3$ có $3$ cách chọn 

Vị trí $4$ có $3$ cách chọn 

Vị trí $5$ có $2$ cách chọn

Vị trí $6$ có $1$ cách chọn

nên có $4.4.1.3.3.2.1=288$ (cách)

Vậy có $480$ cách




#583045 $3\left [ x^{2} \right ]+5\left [ x \right...

Đã gửi bởi tuananh2000 on 19-08-2015 - 11:41 trong Số học

Mình không nhớ rõ về phần nguyên cho lắm,nên lỡ có sai thì thông cảm  ^_^

$PT\Leftrightarrow (3\left [ x \right ]-1)(\left [ x \right ]+2)=0\Leftrightarrow \begin{bmatrix} 3\left [ x \right ]-1=0 & \\ \left [ x \right ]+2=0 & \end{bmatrix}\Leftrightarrow \begin{bmatrix} \left [ x \right ]=\frac{1}{3}(VL) & \\ \left [ x \right ]=-2\Rightarrow -2\leq x< -1 & \end{bmatrix}$

Vậy....

$\left [ x^{2} \right ]=\left [ x \right ].\left [ x \right ]$ ?

P/s: Mình đang phân vân đoạn này




#583039 $3\left [ x^{2} \right ]+5\left [ x \right...

Đã gửi bởi tuananh2000 on 19-08-2015 - 11:24 trong Số học

Giải phương trình : $3\left [ x^{2} \right ]+5\left [ x \right ]=2$




#580176 Topic post ảnh người yêu, bạn gái,...

Đã gửi bởi tuananh2000 on 09-08-2015 - 22:14 trong Góc giao lưu

Sau bao cảm xúc dồn nén bấy lâu ,tối nay em up ngay hình "Gấu" của em

Xinh vậy , mình thích điều này  :D  :like




#576890 Tìm min A = $\frac{xy}{z}+\frac{yz...

Đã gửi bởi tuananh2000 on 30-07-2015 - 21:05 trong Bất đẳng thức và cực trị

Câu 1:Cho $\left\{\begin{matrix} x,y,z>0\\ x^{2}+y^{2}+z^{2} = 1 \end{matrix}\right.$

Tìm min A = $\frac{xy}{z}+\frac{yz}{x}+\frac{xz}{y}$

Câu 2: Cho x,y>1. Tìm min B = $\frac{x^{3}+y^{3}-(x^{2}+y^{2})}{(x-1)(y-1)}$

Câu 1:

$A^{2}=\sum \frac{x^{2}.y^{2}}{z^{2}}+2\sum x^{2}\geq 1+2=3$

Câu 2:

$B=\frac{x^{2}}{y-1}+\frac{y^{2}}{x-1}\geq \frac{2xy}{\sqrt{(x-1)(y-1)}}\geq 8$ ($AM-GM$ dạng mẫu)




#575130 Phổ điểm kỳ thi THPT Quốc Gia 2015

Đã gửi bởi tuananh2000 on 24-07-2015 - 21:39 trong Tin tức - Vấn đề - Sự kiện

Việt Nam ưu tiên nói Tiếng Việt chứ đâu ai dùng Tiếng Anh là mấy  :D




#574534 cm:$\sum \frac{1}{a(3a-1)}\leq 2$

Đã gửi bởi tuananh2000 on 22-07-2015 - 08:41 trong Bất đẳng thức và cực trị

Bài này một ẩn bạn dùng PP $UCT$ đi,  :icon6:




#574529 $3(x^4+y^4+x^2+y^2+2)=2(x^2-x+1)(y^2-y+1)$

Đã gửi bởi tuananh2000 on 22-07-2015 - 08:24 trong Số học

Giải phương trình nghiệm nguyên dương:

a) $(x^3+y^3)+4(x^2+y^2)+4(x+y)=16xy$

b) $3(x^4+y^4+x^2+y^2+2)=2(x^2-x+1)(y^2-y+1)$

c) $x^3+y^3+z^3=(x+y+z)^2$

Spoiler

c) Ta dùng $AM-GM$ 

$\sum x^{3}\leq 3\sum x^{2}$

$\Leftrightarrow \sum x^{2}(x-3)\leq 0$

Hoàn toàn có thể giả sử $x\geq y\geq z> 0 (x;y;z\in N*)$

$\Rightarrow z\leq 3$

Nếu $z=3$ thì $x^{2}(x-3)+y^{2}(y-3)\leq 0$

$\Rightarrow y\leq 3$

Bạn xét các TH để cho $x$ nguyên dương nữa là được

Nếu $z=2$ thì $x^{2}(x-3)+y^{2}(y-3)\leq 4$

$\Rightarrow y\leq 3$

Làm tiếp tục thử

Với TH $z=1$ thì cũng làm vậy




#573168 IMO 2015: Việt Nam xếp thứ 5 toàn đoàn với 2 Vàng, 3 Bạc, 1 Đồng

Đã gửi bởi tuananh2000 on 16-07-2015 - 20:48 trong Tin tức - Vấn đề - Sự kiện

Alex Song - A beautiful mind http://theexonian.co...autiful-mind-2/

Đây là lần thứ 5 đạt HCV 




#569597 $for$ i:=1 $to$ n $do$

Đã gửi bởi tuananh2000 on 03-07-2015 - 09:08 trong Góc Tin học

Ừ...

$1.$-Cho biến $i$ chạy từ $1$ đến $n$:

-Nếu $i$ chia hết cho $2$ thì tăng biến $c$ lên $1$.

$2.$-Giống giống với $1$.

$3.$-Cho biến $d$ bằng $0$.

-Lặp khi $n$ vẫn dương:

      -Lấy phần nguyên của $n$ khi chia cho $2$.

      -Tăng biến $d$.

P/s: Có gì sai yêu cầu đừng oánh nhé!!!

Vd bài 1 có đáp số thời gian là $O(n)$ mà , đây là độ phức tạp của thuật toán mà , bạn giải cụ thể chứ đừng dịch đề  :namtay  :namtay




#569589 $for$ i:=1 $to$ n $do$

Đã gửi bởi tuananh2000 on 03-07-2015 - 08:53 trong Góc Tin học

Mình có học tin tí xíu, nhưng bạn phải nói rõ đề cụ thể thì mình mới hướng dẫn bạn được....

đề là phân tích thời gian thực hiện các đoạn chương trình đó bạn




#569588 CMR: $\frac{a}{c}+\frac{b}{...

Đã gửi bởi tuananh2000 on 03-07-2015 - 08:50 trong Bất đẳng thức và cực trị

giải thích cho mình chỗ 

$\sum \frac{a}{c}+\sqrt[3]{abc}\geq \frac{a+b+c}{\sqrt[3]{abc}}+\sqrt[3]{abc}$

được không bạn???

Ta có $\frac{a}{c}+\frac{a}{c}+\frac{c}{b}\geq \frac{3a}{\sqrt[3]{abc}}$

TT cộng lại là được 




#569567 Bánh canh chém gió về kì thi IMO 2015

Đã gửi bởi tuananh2000 on 03-07-2015 - 07:15 trong Thi HSG Quốc gia và Quốc tế

Bổ sung vào danh sách của anh Toàn anh nguyenthehoan  :icon6:  :icon6:  :icon6:  :icon6:




#569546 CMR: $\sum \frac{a^{2}}{6a^{2...

Đã gửi bởi tuananh2000 on 02-07-2015 - 22:37 trong Bất đẳng thức và cực trị

Cho các số thực a,b,c thỏa mãn a+b+c = 1. CMR : 

$\frac{a^{2}}{6a^{2}-4a+1}+\frac{b^{2}}{6b^{2}-4b+1}+\frac{c^{2}}{6c^{2}-4c+1}\leq 1$

Mình nhầm , mod xóa hộ  :wacko:  :wacko: